Pharm Antiarrhythmics

Approved & Edited by ProProfs Editorial Team
The editorial team at ProProfs Quizzes consists of a select group of subject experts, trivia writers, and quiz masters who have authored over 10,000 quizzes taken by more than 100 million users. This team includes our in-house seasoned quiz moderators and subject matter experts. Our editorial experts, spread across the world, are rigorously trained using our comprehensive guidelines to ensure that you receive the highest quality quizzes.
Learn about Our Editorial Process
| By Chachelly
C
Chachelly
Community Contributor
Quizzes Created: 513 | Total Attempts: 592,897
Questions: 20 | Attempts: 4,537

SettingsSettingsSettings
Pharm Antiarrhythmics - Quiz

In today’s medical quiz, we’ll be looking at antiarrhythmics, which are pharmaceuticals that help to ease abnormal rhythms of the heart, which can come about as a result of a number of different heart conditions. What can you tell us about these kinds of drugs? Let’s take a look.


Questions and Answers
  • 1. 

    Directions: questions 1-6 Match each antiarrhythmic drug with the appropriate description (each lettered option can be selected once, more than once, or not at all): This drug is an anticonvulsant frequently used in case of generalized tonic-clonic seizures.

    • A.

      Adenosine

    • B.

      Amiodarone

    • C.

      Diltiazem

    • D.

      Esmolol

    • E.

      Flecainide

    • F.

      Ibutilide

    • G.

      Lidocaine

    • H.

      Magnesium sulfate

    • I.

      Metoprolol

    • J.

      Mexiletine

    • K.

      Phenytoin

    • L.

      Procainamide

    • M.

      Quinidine

    • N.

      Sotalol

    Correct Answer
    K. Phenytoin
    Explanation
    Phenytoin is an anticonvulsant drug frequently used in the case of generalized tonic-clonic seizures. It is not commonly used as an antiarrhythmic drug, but it can be used in certain cases to treat arrhythmias.

    Rate this question:

  • 2. 

    Directions: questions 1-6 Match each antiarrhythmic drug with the appropriate description (each lettered option can be selected once, more than once, or not at all): This drug acts on acetylcholine sensitive K+ channels

    • A.

      Adenosine

    • B.

      Amiodarone

    • C.

      Diltiazem

    • D.

      Esmolol

    • E.

      Flecainide

    • F.

      Ibutilide

    • G.

      Lidocaine

    • H.

      Magnesium sulfate

    • I.

      Metoprolol

    • J.

      Mexiletine

    • K.

      Phenytoin

    • L.

      Procainamide

    • M.

      Quinidine

    • N.

      Sotalol

    Correct Answer
    A. Adenosine
    Explanation
    Adenosine acts on acetylcholine sensitive K+ channels.

    Rate this question:

  • 3. 

    Directions: questions 1-6 Match each antiarrhythmic drug with the appropriate description (each lettered option can be selected once, more than once, or not at all): This drug is the most effective antiarrhythmic agent for both supraventricular and ventricular arrhythmias

    • A.

      Adenosine

    • B.

      Amiodarone

    • C.

      Diltiazem

    • D.

      Esmolol

    • E.

      Flecainide

    • F.

      Ibutilide

    • G.

      Lidocaine

    • H.

      Magnesium sulfate

    • I.

      Metoprolol

    • J.

      Mexiletine

    • K.

      Phenytoin

    • L.

      Procainamide

    • M.

      Quinidine

    • N.

      Sotalol

    Correct Answer
    B. Amiodarone
    Explanation
    Amiodarone is the most effective antiarrhythmic drug for both supraventricular and ventricular arrhythmias. It is a class III antiarrhythmic medication that works by prolonging the duration of action potential and refractory period in cardiac tissues. It is used to treat a wide range of arrhythmias, including atrial fibrillation, ventricular tachycardia, and ventricular fibrillation. Amiodarone has a broad spectrum of activity and is considered a first-line therapy for many types of arrhythmias due to its efficacy and safety profile.

    Rate this question:

  • 4. 

    Directions: questions 1-6 Match each antiarrhythmic drug with the appropriate description (each lettered option can be selected once, more than once, or not at all): This drug blocks inactivated (but not activated) Na+ channels

    • A.

      Adenosine

    • B.

      Amiodarone

    • C.

      Diltiazem

    • D.

      Esmolol

    • E.

      Flecainide

    • F.

      Ibutilide

    • G.

      Lidocaine

    • H.

      Magnesium sulfate

    • I.

      Metoprolol

    • J.

      Mexiletine

    • K.

      Phenytoin

    • L.

      Procainamide

    • M.

      Quinidine

    • N.

      Sotalol

    Correct Answer
    B. Amiodarone
    Explanation
    Amiodarone is the correct answer because it is a class III antiarrhythmic drug that blocks inactivated (but not activated) Na+ channels. It also has additional effects on other ion channels, making it effective in treating a wide range of arrhythmias.

    Rate this question:

  • 5. 

    Directions: questions 1-6 Match each antiarrhythmic drug with the appropriate description (each lettered option can be selected once, more than once, or not at all): This drug blocks both beta receptors and K+ channels

    • A.

      Adenosine

    • B.

      Amiodarone

    • C.

      Diltiazem

    • D.

      Esmolol

    • E.

      Flecainide

    • F.

      Ibutilide

    • G.

      Lidocaine

    • H.

      Magnesium sulfate

    • I.

      Metoprolol

    • J.

      Mexiletine

    • K.

      Phenytoin

    • L.

      Procainamide

    • M.

      Quinidine

    • N.

      Sotalol

    Correct Answer
    N. Sotalol
    Explanation
    Sotalol is the correct answer because it is a non-selective beta blocker that also blocks potassium channels. This dual mechanism of action helps to slow down heart rate and prolong the duration of the action potential, making it effective in treating certain types of arrhythmias.

    Rate this question:

  • 6. 

    Directions: questions 1-6 Match each antiarrhythmic drug with the appropriate description (each lettered option can be selected once, more than once, or not at all): This drug may be used for the therapy of malaria

    • A.

      Adenosine

    • B.

      Amiodarone

    • C.

      Diltiazem

    • D.

      Esmolol

    • E.

      Flecainide

    • F.

      Ibutilide

    • G.

      Lidocaine

    • H.

      Magnesium sulfate

    • I.

      Metoprolol

    • J.

      Mexiletine

    • K.

      Phenytoin

    • L.

      Procainamide

    • M.

      Quinidine

    • N.

      Sotalol

    Correct Answer
    M. Quinidine
    Explanation
    Quinidine is an antiarrhythmic drug that may also be used for the therapy of malaria. It is effective in treating certain types of malaria, particularly those caused by Plasmodium falciparum. Quinidine works by interfering with the DNA replication of the malaria parasite, ultimately leading to its death. In addition to its antimalarial properties, Quinidine is also used to treat certain types of cardiac arrhythmias.

    Rate this question:

  • 7. 

    A 51-year-old woman was found to have a heart rhythm of 110 bpm during a control visit. The woman, suffering from atrial flutter, had been receiving quinidine for two weeks, after a successful electrical cardioversion. The physician thought that her tachycardia was caused by quinidine. Which of the following actions best explains the mechanism of this adverse effect of quinidine?

    • A.

      Blockade of muscarinic receptors

    • B.

      Activation of alpha-1 receptors

    • C.

      Stimulation of arterial baroreceptors

    • D.

      Activation of Ca channels

    • E.

      Activation of potassium channels

    Correct Answer
    A. Blockade of muscarinic receptors
    Explanation
    Learning objective: describe the mechanism of quinidine-induced paradoxical tachycardia.
    Answer: A
    Quinidine is still sometimes used to maintain the normal sinus rhythm after cardioversion, but it
    can cause paradoxical tachycardia, due to its antimuscarinic action on the heart, which increases
    AV conduction.
    B) Quinidine blocks, not activates, alpha-1 receptors so causing peripheral vasodilation. Even if
    the decreased blood pressure usually causes reflex tachycardia in normal persons, this is not the
    case in patients with atrial flutter since the SA node is no longer the main pacemaker.
    C) By causing vasodilation quinidine actually decreases the stimulation of arterial baroreceptors
    D, E) Quinidine can block, not activates, both calcium and potassium channels.

    Rate this question:

  • 8. 

    A Purkinje fiber was isolated from an animal heart and placed in a recording chamber. Action potentials were recorded before and after a low dose of quinidine was added to the perfusate. Which of the following was an electrophysiological action of quinidine on this preparation?

    • A.

      Decreased action potential duration

    • B.

      Increased slope of phase 3

    • C.

      Decreased slope of phase 0

    • D.

      Decreased effective refractory period

    • E.

      Increased slope of phase 4

    Correct Answer
    C. Decreased slope of phase 0
    Explanation
    Learning objective: describe the electrophysiological actions of quinidine.
    Answer: C
    Blockade of activated sodium channels is the distinctive feature of class Ia antiarrhythmic drugs.
    The slope of phase zero of the cardiac action potential in normal atrial, Purkinje and ventricular
    cells is dependent on sodium current. The higher the number of sodium channels that are in the
    open state, the higher the sodium current and the steeper the slope of phase zero. By blocking
    activated sodium channels (the number of channels that are blocked is dose-dependent) the
    sodium current will be less intense and the slope of phase zero decreases.
    A, B, D) By blocking K+ channels quinidine slow down repolarization and therefore action
    potential duration is increased, the slope of phase 3 is decreased and the effective refractory
    period is increased.
    E) The slope of phase 4 is related to automaticity (the steeper the slope the higher the
    automaticity). Most antiarrhythmic drugs decrease the slope of phase 4, unless toxic doses are
    given.

    Rate this question:

  • 9. 

    A 63-year-old man was admitted to the hospital because of fever, chills, and profuse epistaxis. The man, recently diagnosed with a supraventricular arrhythmia, had started an antiarrhythmic drug the day before. Lab results on admission disclosed a platelet count of 50,000/mm . Which of the following drugs most 3 likely caused the patient’s disorder?

    • A.

      Lidocaine

    • B.

      Verapamil

    • C.

      Quinidine

    • D.

      Adenosine

    • E.

      Sotalol

    • F.

      Diltiazem

    Correct Answer
    C. Quinidine
    Explanation
    Learning objective: describe the main adverse effects of quinidine.
    Answer: C
    More than 100 drugs have been implicated to cause thrombocytopenia, but quinidine and
    heparin are the two drugs that have the strongest association with this disorder. Quinidineinduced
    thrombocytopenia appears to be immunologically mediated and usually develops within
    12-24 hours from ingestion of the drug by a sensitized individual.
    A, B, D, E, F) The risk of drug-induced thrombocytopenia with these drugs is quite low.

    Rate this question:

  • 10. 

    A 24-year-old man presented to the emergency room with a chief complain of palpitations for three hours. He had experienced no prior symptoms and had no significant past medical history. Vital sings were: pulse 190 bpm, blood pressure 100/60 mm Hg, respiratory rate 14/min. An ECG showed a picture compatible with Wolff- Parkinson-White syndrome. An appropriate therapy was instituted which included an IV administration of a drug. Which of the following drugs was most likely given?

    • A.

      Digoxin

    • B.

      Verapamil

    • C.

      Propranolol

    • D.

      Procainamide

    • E.

      Nifedipine

    Correct Answer
    D. Procainamide
    Explanation
    Learning objective: describe the therapeutic uses of procainamide.
    Answer: D
    Wolff-Parkinson-White (WPW) is a pre-excitation syndrome in which there is an accessory bypass tract (known as a Kent bundle) connecting the atria to the ventricle. An impulse can travel
    down this pathway and excite the ventricle before the expected regular impulse through the AV
    node. Hence the term pre excitation. WPW syndrome can occur in children and adult without
    overt cardiac disease. The true incidence is unknown but varies in different reports from 0.1 to 3
    per 1000 ECGs. About 20-30% of all supraventricular tachycardias are associated with WPW
    syndrome.
    DC cardioversion represents the treatment of choice for WPW syndrome. Medical therapy must
    be used cautiously because it may unexpectedly increase the ventricular rate. Of the FDA
    approved drugs, procainamide, which prolongs the refractory period of the accessory pathways,
    is considered by many the drug of choice.
    A, B, C) Digoxin, calcium channel blockers and beta-blockers may be dangerous by blocking the
    AV node and redirecting impulses down the bypass tract.
    E) Nifedipine has no direct action on the heart and the reflex tachycardia it induces may worsen
    the arrhythmia.

    Rate this question:

  • 11. 

    A 47-year-old woman suffering from sustained ventricular tachycardia had been receiving mexiletine for one month. In the abnormal pacemaker cells of this patient the drug most likely decreased which of the following electrophysiological parameters of the heart?

    • A.

      Refractoriness

    • B.

      Slope of phase 4

    • C.

      Action potential duration

    • D.

      Length of phase 3

    • E.

      Diastolic interval

    Correct Answer
    B. Slope of phase 4
    Explanation
    Learning objective: describe the electrophysiological actions of mexiletine.
    Answer: B
    The typical action of most antiarrhythmic drugs is the decreased slope of phase 4 in pacemaker
    cells, which means a decrease in automaticity.
    A, C) Mexiletine is a class 1b drug. These agents can decrease action potential duration (and so
    refractoriness) in normal cardiac cells but they increase it in abnormal cells.
    D) Phase 3 is due to movement of calcium across cardiac cell membrane. Class 1b drugs do not
    affect calcium movement.
    E) Antiarrhythmic drugs tend to decrease cardiac rate and therefore the interval between two
    action potentials (i.e. the diastolic interval) is increased, not decreased.

    Rate this question:

  • 12. 

    A 47-year-old man, admitted to the coronary unit because of a myocardial infarction, developed increasing ectopic beats followed by sustained tachycardia. The ECG showed a frequency of 175 bpm, wide QRS complexes and atrioventricular dissociation. He was given an IV infusion of an antiarrhythmic drug which restored the normal sinus rhythm, but one hour later the patient showed increased agitation, loss of coordination, confusion, slurred speech, nystagmus, trembling and muscle twitching. Which of the following drug was most likely administered?

    • A.

      Lidocaine

    • B.

      Phenytoin

    • C.

      Sotalol

    • D.

      Mexiletine

    • E.

      Verapamil

    • F.

      Digoxin

    Correct Answer
    A. Lidocaine
    Explanation
    Learning objective: describe the main adverse effects of lidocaine
    Answer: A
    Lidocaine is a drug of choice for ventricular arrhythmias associated with myocardial infarction and
    the patient’s tachycardia was most likely ventricular in nature, since atrioventricular dissociation
    was noted. Patient’s symptoms and sings are typical CNS effects of a high dose of lidocaine.
    B, C) High doses of phenytoin or sotalol do not cause the pattern of symptoms and signs
    exhibited by the patient.
    D) Mexiletine is a lidocaine-like drug that is used only orally to prevent recurrence of ventricular
    tachycardia.
    E, F) These drugs are not used, and may be even contraindicated, in case of ventricular
    tachycardia.

    Rate this question:

  • 13. 

    A 27-year-old cocaine addict was admitted to the emergency department after an intravenous injection of a high dose of cocaine. Physical examination showed an agitated and uncooperative patient with normal sensorium an purposeless psychomotor activity. Vital signs on admission were: blood pressure 190/100 mm Hg, pulse 180 beats/min, respirations 20/min. An appropriate therapy was initiated, which included the parenteral administration of a drug. Which of the following drugs was most likely given?

    • A.

      Procainamide

    • B.

      Labetalol

    • C.

      Digoxin

    • D.

      Flecainide

    • E.

      Verapamil

    Correct Answer
    B. Labetalol
    Explanation
    Learning objective: describe the antiarrhythmic use of labetalol. Answer: B Most symptoms of cocaine overdose are due to catecholamines since the drug inhibits their reuptake into adrenergic terminals. Beta-blockers are usually contraindicated in the acute setting of cocaine toxicity since they decrease cardiac pumping activity in the presence of an increase total peripheral resistance. Labetalol provides both alpha and beta blockade and can be used in catecholamine-mediated hypertension and tachycardia even if it is not considered a first-line treatment. A, D) These drugs are sometimes used to treat ventricular arrhythmias but in this case a alphabeta blocker is preferred (see explanation above) C, E) These drugs are contraindicated in ventricular arrhythmias.

    Rate this question:

  • 14. 

    A 54-year-old woman complained to her physician of palpitations, insomnia, diarrhea and increased sweating for the last three weeks. Physical examination revealed a patient in moderate distress with mild hand tremors and exophthalmos. Vital sings were: pulse 122 bpm, blood pressure 146/62 mm Hg, respiratory rate 18/min. An ECG showed atrial tachycardia. Which of the following drugs would be most appropriate to treat the patient’s arrhythmia?

    • A.

      Quinidine

    • B.

      Amiodarone

    • C.

      Verapamil

    • D.

      Propranolol

    • E.

      Digoxin

    Correct Answer
    D. Propranolol
    Explanation
    Learning objective: describe the uses of propranolol in hyperthyroidism.
    Answer: D
    The patient presents with classical symptoms of hyperthyroidism. When a tachyarrhythmia is
    associated with hyperthyroidism beta-blockers remain the drugs of choice, since many symptoms
    of this disease are associated with sympathetic activation.
    A, B, C, E) All the other listed drugs can be used to treat supraventricular arrhythmias, but in the
    present case a beta-blocker is preferred (see explanation above)

    Rate this question:

  • 15. 

    A 24-year-old man was admitted to the emergency room with severe dyspnea and chest pain. Auscultation of the heart showed an ejection-type murmur and chest x-ray disclosed a massive cardiomegaly. His father was reported to have died from a similar problem that was likely related to a gene defect. The ECG showed atrial fibrillation. An appropriate therapy was ordered. Which of the following drugs was most likely given?

    • A.

      Procainamide

    • B.

      Quinidine

    • C.

      Flecainide

    • D.

      Propranolol

    • E.

      Nitroglycerin

    Correct Answer
    D. Propranolol
    Explanation
    Learning objective: describe the use of beta-blockers for atrial fibrillation associated with
    hypertrophic congenital hypertrophic cardiomyopathy.
    Answer: D
    The patient is likely affected by congenital hypertrophic cardiomyopathy, a disorder usually
    inherited in an autosomal dominant pattern. Classic symptoms of the disease are angina (a
    limited coronary blood flow reserve is present in hypertrophied states) and diastolic heart failure
    (due to the reduced compliance of the hypertrophied left ventricle). Beta-blockers without partial
    agonist activity (and calcium channel blockers that act on the heart, like verapamil and diltiazem)
    are drugs of choice in this disease because they decrease cardiac contractility and increase
    diastolic ventricular filling (by causing bradycardia). Beta-blocker are also indicated in atrial
    fibrillation and therefore, in the present case, they are useful for both conditions.
    A, B, C) All the listed drug can be used in atrial fibrillation, but in this case beta-blockers are
    preferred (see explanation above)
    E) Nitrates are contraindicated in hypertrophic cardiomyopathy since they reduce preload, which
    is already low because of the reduced volume and compliance of the left ventricle)

    Rate this question:

  • 16. 

    A 58-year-old man was admitted to the hospital because of dyspnea, a nonproductive cough and fever. The man, suffering from sustained ventricular tachycardia, had been receiving an antiarrhythmic drug for one month. Chest x-ray showed diffuse bilateral infiltrates. Bacterial, fungal and viral cultures were negative. Which of the following drugs most likely caused the patient’s pulmonary disorder?

    • A.

      Flecainide

    • B.

      Mexiletine

    • C.

      Amiodarone

    • D.

      Sotalol

    • E.

      Procainamide

    Correct Answer
    C. Amiodarone
    Explanation
    Learning objective: describe the main adverse effects of amiodarone.
    Answer: C
    The sings and symptoms of the patient indicate a pulmonary disorder. Antimicrobial infection is
    unlikely and diffuse bilateral infiltrated are consistent with pulmonary fibrosis. Amiodaroneinduced
    pulmonary fibrosis is the most serious adverse effect of the drug. It occurs in 5-10% of
    the population with a mortality of 5-10% of those affected.
    A, B, D, E) All the other listed drugs may be used for sustained ventricular tachycardia
    prophylaxis, but they do not cause pulmonary fibrosis.

    Rate this question:

  • 17. 

    A 44-year-old man complained to his physician of joint pains in elbows and knees and of an unusual mask-like rash over his face. The man, who was suffering from Wolff- Parkinson-White syndrome, had been receiving an antiarrhythmic drug for one month. Discontinuation of the drug caused the symptoms to abate. Which of the following drugs did the patient most likely take?

    • A.

      Quinidine

    • B.

      Lidocaine

    • C.

      Amiodarone

    • D.

      Adenosine

    • E.

      Procainamide

    • F.

      Ibutilide

    Correct Answer
    E. Procainamide
    Explanation
    Learning objective: describe the lupoid syndrome caused by procainamide.
    Answer: E
    The symptoms of the patient are consistent with drug-induced lupus (also called lupoid
    syndrome) an autoimmune disorder that is quite similar to the idiopathic systemic lupus
    erythematosus. The most common agent reported to cause the disorder are procainamide
    (about one third of the patients taking the drug over one year period) and hydralazine. Other
    drugs that can cause drug-induced lupus include chlorpromazine, isoniazid, methyldopa,
    quinidine, sulfonamides and penicillamine. The drug-induced lupus syndrome onset can
    occur as soon as one month of therapy, like in the present case. Unlike the idiopathic lupus,
    drug-induced lupus typically improves rapidly after the discontinuation of the drug.
    A) Quinidine can cause lupus but is not used to treat Wolff-Parkinson-White syndrome.
    B, C, D, F) The risk of drug-induced lupus with these drugs is quite low.

    Rate this question:

  • 18. 

    A 56-year-old woman was admitted to the intensive care unit because of chest palpitations for three hours. Vital sings on admission were: blood pressure 96/70 mm Hg, heart rate 210 bpm, respiration 15 breaths/min. Physical examination showed a neck pulsation corresponding to the heart rate . An ECG indicated atrial fibrillation with wide QRS. A diagnosis of atrioventricular nodal reentrant tachycardia was made and amiodarone was given IV, but fifteen minutes later the heart rate was still 180 bpm. Another drug was tried IV. Which of the following drugs was most likely administered?

    • A.

      Lidocaine

    • B.

      Mexiletine

    • C.

      Phenytoin

    • D.

      Nifedipine

    • E.

      Nitroglycerin

    • F.

      Procainamide

    Correct Answer
    F. Procainamide
    Explanation
    Learning objective: choose the appropriate antiarrhythmic drug for a patient with an
    atrioventricular nodal tachycardia resistant to amiodarone.
    Answer: F
    Atrioventricular nodal reentrant tachycardia is the most common form of supraventricular
    reentrant tachycardia and is often due to a functional division of the AV node into two pathways
    with different conduction characteristics. One is a fast conducting pathway and the other is a
    slower conducting pathway. Neck pulsations are unique to this disorder. Medical treatment is
    directed at slowing conduction in both pathways. Typical nodal blocking agents like betablockers,
    Ca channel blockers, digoxin or adenosine are most effective on the slow pathway but
    amiodarone or procainamide are preferred since they can affect both pathways.
    A, B, C, D, E) All these drugs are minimally effective in supraventricular arrhythmias.

    Rate this question:

  • 19. 

    A 53-year-old woman with a history of major depression was brought to the emergency room by her husband because of lethargy. Her medications included amitriptyline and the husband referred that he found an empty bottle of the medication in her room. Physical examination showed a lethargic, oriented patient with pulse 135 bpm, blood pressure 113/64, respiratory rate 22/min. The ECG showed a ventricular arrhythmia with widened QRS complexes. An appropriate therapy was instituted and an antiarrhythmic drug was prescribed. Which of the following antiarrhythmic drugs would be absolutely contraindicated for this patient?

    • A.

      Mexiletine

    • B.

      Lidocaine

    • C.

      Quinidine

    • D.

      Phenytoin

    • E.

      Propranolol

    Correct Answer
    C. Quinidine
    Explanation
    Learning objective: identify the main contraindications of quinidine.
    Answer: C
    Class Ia and Ic antiarrhythmic drugs are absolutely contraindicated in case of poisoning with
    tricyclic antidepressants as they share cardiotoxic and anticholinergic side effects with these
    drugs. Moreover antiarrhythmic drugs that increase action potential duration are generally
    contraindicated in patients with widened QRS complexes.
    A, B) These drugs are indicated, not contraindicated, in case of ventricular arrhythmias.
    D) Phenytoin is not contraindicated in ventricular arrhythmias even if its efficacy remains
    controversial, except when arrhythmias are due to digitalis toxicity, .
    E) Beta-blockers might be effective in treating tricyclic-induced arrhythmias even if they are not
    drugs of choice because they can cause bradycardia and worsen tricyclic-induced hypotension.

    Rate this question:

  • 20. 

    A 54-year-old woman was admitted to the hospital because of an episode of dizziness and near syncope. Medical history of the patient was significant for urinary tract infection, presently treated with ciprofloxacin. Few days ago she was diagnosed with atrial fibrillation and started a treatment with sotalol, one tablet daily. An ECG strip recorded by a Holter monitor during another episode of near syncope clarified the diagnosis. Which of the following disorders did the patient most likely suffer from?

    • A.

      Premature ventricular contractions

    • B.

      Atrial fibrillation

    • C.

      Second degree AV block

    • D.

      Polymorphic ventricular tachycardia

    • E.

      Atrial tachycardia

    • F.

      Ventricular fibrillation

    Correct Answer
    D. Polymorphic ventricular tachycardia
    Explanation
    Learning objective: describe the main adverse effects of sotalol.
    Answer: D
    The electrocardiographic hallmark of polymorphic ventricular tachycardia is a long Q-T interval.
    Prolongation of Q-T interval indicates prolongation of action potential duration, which is related to
    a decreased outward potassium current during the phase 3 of the action potential. Long Q-T
    interval is present prior to the onset of tachycardia and is due to hereditary or acquired potassium
    channel defects. Hereditary Q-T syndromes are due to some rare mutations of the genes
    encoding potassium channels. However the importance of the disorder in everyday practice is
    related to its provocation by diseases or drugs that prolong Q-T interval. Diseases include
    hypothyroidism, subarachnoid hemorrhage, myocarditis, hypokalemia and hypomagnesemia.
    Drugs include class Ia and class III antiarrhythmic drugs, tricyclic antidepressants, neuroleptics,
    some antihistamines, macrolide antibiotics and quinolones. All these drugs are able to increase
    action potential duration by blocking or modifying potassium channels. High doses of these drugs
    can trigger polymorphic ventricular tachycardia in patients at risk.
    Sotalol is the only beta-blocker that can block potassium channels ( a property not related to beta
    receptor blockade) and it can cause polymorphic ventricular tachycardia. Moreover the patient
    was taking a quinolone and so was already at risk of developing the disorder.
    A, C, D, E, F) (see explanation above).

    Rate this question:

Quiz Review Timeline +

Our quizzes are rigorously reviewed, monitored and continuously updated by our expert board to maintain accuracy, relevance, and timeliness.

  • Current Version
  • Mar 22, 2023
    Quiz Edited by
    ProProfs Editorial Team
  • Jul 08, 2012
    Quiz Created by
    Chachelly
Back to Top Back to top
Advertisement
×

Wait!
Here's an interesting quiz for you.

We have other quizzes matching your interest.